You are on page 1of 39

? The patient, 52 years, complains of a retrosternal burning pain, appearing at physical loading and disappearing after it.

Objectively: a condition of the patient is satisfactory, pulse beat - 86 times a minute, during auscultation the tones of heart are rhythmical, are muffled. The BP - 130/80. On the electrocardiogram a pathological changes are not revealed. What is your preliminary diagnosis? +Stenocardia stable on exertion -Stenocardia unstable -Myocarditis -Q miocardial infarction -Cardiomyopathy ? Patient p, 45y.o. was delivered by an ambulance with complaints of pain in the epigastrium, vomiting at the top of pain with impurity of blood; heartburn, weakness, dizziness, black stool in the morning. He has been ill for 10 years, nutrition is irregular. What complication has developed in the patient? +Gastric bleeding -Intestinal bleeding -Bleeding from esophageal veins -Bleeding from hemorrhoidal veins -Thrombocytopenia ? A 65 year old patient was admitted in the hospital with a complains of fatigue,feeling of cotton likelegs, loss of sensitivitiy of toes. On examination pale of the skin,subecteric sclera.. Heart sounds of low intensity,systolic murmur heard over the heart area ,liver +2cm,painless,spleen not enlarged.Blood analysis: Hb 58mg/l, E- 1.3 x10 12/l, color index-1.3, reticulocytes-0.2\%. ESR 30mm/hr, megaloblasts-2:100, macrocytosis. Jolly bodies, ebots ring. Most informative method of investigation? +Serum B-12 and folic acid level -Endoscopic examination of the stomach -X ray of abdomen -Osmotic resistance of erythrocytes -Iron content of serum ? The 63y.o. patients complaints are dyspnea during physical exercise, palpitation and swelling of the ankles. He had a transmural myocardial infarction a year ago. Objectively: T 36.5(C, BR 22 pm, Ps 96 bpm, AP 110/70. Acrocyanosis. Vesicular respiration is evident; it is raised in the lower areas with single faint rales. Left cardiac bound is 1.5cm inward of lin. medioclavicularis. Cardiac tones raised, rhythm is correct, Liver 2cm below arcus costalis. What is the mechanism of tachycardia? -Hypertrophy of myocardium +Raised function of parasympathetic nervous system -Frank-Starlings mechanism -Vasoconstriction -Activation of renin-angiotensine-aldosterone system ? A 56y.o. teacher consulted a department therapist about a corresponding diet. Complaints: frequent constipations [defecation once in 2-3 days], flatulence and unpleasant aftertaste; blunt pain in the lower half of abdomen after intake of ice cream or soda drinks. At 17-20 she was treated for duodenal ulcer. Objectively: height 164cm, weight 62kg. Abdomen is oval, palpation doesnt detect pathological changes. What diet is necessary for prevention of transition of chronic disease into acute? -Table #4 by Pevzner +Table #4 by Pevzner -General table
1

-Table #2 by Pevzner -Table #9 by Pevzner ? A 50yrs ols male has had precaridal pain for 4 hrs. on examintion his BP 110/80mmhg.pulse 10beats /mint anf respiratory rate is 206/min. His ECG maked S-T segement elvation and left ventricle ectopies.The initial therepeutoc modilaties in his case would include? -Lignocanineand streptokineases +Lignocaine,streptokinaseand morphine -Morphineand dobtutamine -Streptokineases and morphine -Nitoglcine +Digoxine ? A 60 year old woman mother of 6 children developed sudden onset of upper abdominal pain radiating to back,associtated with nausea,vomiting ,fever and chills.She noticed yellow discoloration of her sclera and skin.On physical examination the patient was found to be febril with temperature of 38.9C, along with right upper quadrant tenderness? -Infectious hepatitis -Hemoytic jaundice -Choledochal cyst -Ascariasis +Choledocholithiasis ? A 40 year old male pesents with headache and convulisons.His blodd pressure is 120\140 mmhg. Fundus examination revelas papillooedema.Which of the following drug examination will be most suitable for this patient? +Nitropruside+Furosemide -Diazoxide+trimaterence -Nitroprusside+Triamterence -Resaprine -Captopril ? A 75 year old man who had developed diabetes within last 6 months was found to be jaundiced.He has remained asymptomatic,expect for weight loss about10 pounds in 6 months.On physical examination he is found to have a non- tender ,globular ,right upper quadrant mass that moves with respiration.A CT scan shows enlargement of the head of the pancreas, with no filling defects in the liver? -Choledocholithiasis -Metastatic disease of the liver -Pancreatitis +Carcinoma of the head of the pancreas -Hemolytic jaundice ? Spherical skin defect, size: under 3 cm in the diameter, with gentle margins, necrotic profound fondues, easily, bleeding has been revealed on the patients skin of the lateral surface of the lower part one-third part of the left crush. What morphogical element can we think about? -Profound pustule -Erosion -Fissure +Ulcer -Superficial pustule ? A 55 years old man complaints with chest pain for a duration of 25 min. With radiation to the left side which took place 3 days ago at rest and dissapeared without any medical treatment. The last day attack took place for 5-6 times per day with increases in pain intensity. The results of laboratory test: Troponin-0,17ng/mg, L-5,6
2

(109 /l, alaninaminotransferase-100 mmol/l, Lactat Dehydrohenase-350 mmol/l. Which disease do you suggested? -Stable sternocardia -Myocarditis -Q-Myocardial Infarction -Cardiomyopathy +Non-stable sternocardia ? A 20y.o. student fell ill with acute glomerulonephritis. He was ill with angina a week ago. Objectively: skin and mucous layers pallor, edema of face, limbs and trunk. AP 180/110. Urogram: protein 6.6 g/l, RBC 100-120 in f/v, hyaline cylinders 2-3 in f/v, oligoanuria. What infection had in important part in development of disease? -Intestinal bacillus -Virus -Staphylococcus -Blue-purulent bacillus +Streptococcus ? The patient has peptic ulcer of stomach with frequent recurrences. Courses of anti-recurrent therapy must be? +Once in 3 months -Twice a year -Each year -Each month -After each recurrence ? A 65y.o. patient is in the reanimation department with transmural myocardial infarction. He complains of abdominal pain, flatulence and constipation. Objectively: Abdomen is algestic in the region of colon, with continent inflation; its lower part is full. What is the main mechanism of this condition? +Hypodynamia -Failure of nervous regulation -Non-adequate alimentation -Colon abnormality -Failure of aqueous-saline exchange ? A patient complains of resting dyspnea, high temperature and chest pain. Objectively: The right half of chest is behind in the act of respiration. During percussion a blunt sound is evident. Sounds are absent in lungs during auscultation. X-ray: a homogenous opacity 2/3 of right lung. The most informative for diagnosis is? -Bronchoscopy -Bronchography -Pneumotachometria -Spirography +Punction of pleural cavity ? A 62 y.o. patient is ill with chronic obstructive bronchitis. He has smoked for 20 years. Point out the nonmedicamentous prevention of recurrences in this pathology? -Training of respiratory muscles -Refusal from smoking +All measures -Optimization of microclimate -Balanced alimentation
3

? A 53y.o. woman is suffering from essential arterial hypertension of the I degree combined with obesity of the II degree. Which of the following methods assist an accurate decrease of EAH progression risk? -Psychotherapy -Physical therapy +Therapeutic diet -Rodone baths -Collar zone massage ? A 34y.o. patient has been admitted to clinic with complaints of heaviness in the right subrib, nausea and emesis after intake of fried pork; dark urine and light feces. He was ill with viral hepatitis half a year ago, but wasnt on a diet. Objectively: Scleras are subicteric. Abdomen is with continent inflation. Liver is +2cm of costal arc margin on lin. medioclavicularis dextra. Its surface is smooth and it has an elastic consistence. What disease would you think about? +Chronic hepatitis -Cirrhosis of liver -Active hepatitis -Chronic cholecystitis -Chronic pancreatitis ? A 40 year pateint of rhenmatic heart disease complains of anoxeria,weaknessand loss of weigth,breathlessnesss and swelling of feet . on examination temp 39C pulse is 100/mint .Asucultation distolic murmur in the mitral area. Petchical lesion round clavicle and spleen was palpable, Tooth extraction one month ago? -Mital stenosis -Recurrence of rheumatic fever -Throbocytopenia purpure +Subacute bacteria endocardidtis -Aortic stenosis ? If a 55year male presents with normotension, nephroticsyndrome, azotemia, collateral abdominal veins with upward flow and there is history of gross, painless haematuria 6 months back, the mostly likely diagnosis is? -Amyloidosis -Papillary necrosis -Periarteritis +Hypernephroma -Liver Cirrhosis ? A 70 yrs old women is admitted to the hospital with icreasiing fatigabality and dyspone for the several month. Physical examintaion heat rate 100/min regular rhythm and BP of 150/100mmhg engorged neck veins ,third heart sound and rales over lung fields.Skigram on chest show and prominent vascular markings .This patient is likely to be having? -Adult respiratory distress syndrome -A trial fribrillation -Rebound hypertensive crisis -Pulumonary edema +Congestive cardiac failure ? A 45yrs old executive who is a heavy smoker, had severe retrostrenal discomfort while going to the toilet at 7.00 a.m ECG done immediately showed that S-T segement elevation in the infrrioe leads which normalised
4

within an hour. This likely mostly diagnosis is? +Prinzmetal s angina -Acute myocardial infraction -Dissecting aneurysm of arota -Acute pericarditis -Stable angina pectilors ? A 50 year old patient previously in good health and sudden onset of right anterior chest pain without chills,feveror cough.On examintion he has tanchypneic.Over the right lower rib cage there was an inspiratory rub.Chest X ray was negative.Which statement is correct? -Bronchogenic carcinoma is most likely -Pneumococcal pneumonia is mostly ans penicilline should be given +Pulmonary embolus must be cosidered promptly -Bronchoscopy needs to be done promptly -Viral pnenumonia is most likely diagnosis ? A 43 year old man begins to wheeze.There is no way to tell whether this is bronchial asthma or cardiac asthmaThe best drug to be use would be? -Salbutarole -Digitalis -Cortisone -Epinephrine +Aminophylline ? A middle aged patient presents with a short history of hight fever and rigors. On examination the pulse rate is 140/min and there are signs of toxaemia. Chest examination shows restriction of movement over left infraclavicular region and there is dullness on percussion. Vocal resonance is increased with low pitched bronchial breath sounds over the above area. The mostly likely diagnosis? -Plerual effusion -Lung abscess -Bronchial pneumonia +Lobar pneumonia -Acute bronchitis ? A 48 year-old man presnets with arthralagia, abdominal pain, diarrhoea, progressive weight loss, low-grade fever, confusion,memoryloss and opthalamoplegia. Investigation reveal steatorrhoea, imparied xyloseobsorption, abnormal samll- bowel X rays hypoalbuminaemia and anemia .The mostly likely diagnosis? -Chronic enterocolitis -Coeliac disease -Inflammatory bowel disease -Tropical sprue +Whipple s disease ? A 70-year-old female with mild dementia complains of unilateral groin pain. There is some limitation of motion in the right hip. Which of the following is the most appropriate first step in evaluation? -CBC and erythrocyte sedimentation rate -Rheumatoid factor +X-ray of right hip -Bone scan
5

? A 60-year-old female complains of dry mouth and a gritty sensation in her eyes. She states it is sometimes difficult to speak for more than a few minutes. There is no history of diabetes mellitus or neurologic disease. The patient is on no medications. On exam, the buccal mucosa appears dry and the salivary glands are enlarged bilaterally. Which of the following is the best next step in evaluation? -Lip biopsy +Schirmer test and measurement of autoantibodies -IgG antibody to mumps virus -Use of corticosteroids -Administration of a benzodiazepine ? A 40-year-old male complains of exquisite pain and tenderness in the left ankle. There is no history of trauma. The patient is taking a mild diuretic for hypertension. On exam, the ankle is very swollen and tender. There are no other physical exam abnormalities. Which of the following is the best next step in management? -Begin colchicine and broad-spectrum antibiotics +Perform arthrocentesis -Begin allopurinol if uric acid level is elevated -Obtain ankle x-ray to rule out fracture -Apply a splint or removable cast ? A 50-year-old patient with long-standing chronic obstructive lung disease develops the insidious onset of aching in the distal extremities, particularly the wrists bilaterally. There is a 10-lb weight loss. The skin over the wrists is warm and erythematous. There is bilateral clubbing. Plain film is read as periosteal thickening, possible osteomyelitis. Which of the following is the most appropriate management of this patient? +Obtain chest x-ray -Start ciprofloxacin -Aspirate both wrists -Begin gold therapy -Obtain erythrocyte sedimentation rate ? A patient with low-grade fever and weight loss has poor excursion on the right side of the chest with decreased fremitus, flatness to percussion, and decreased breath sounds all on the right. The trachea is deviated to the left. Which of the following is the most likely diagnosis? -Pneumothorax +Pleural effusion -Consolidated pneumonia -Atelectasis -Chronic obstructive lung disease ? A 40-year-old alcoholic develops cough and fever. Chest x-ray shows an air-fluid level in the superior segment of the right lower lobe. Which of the following is the most likely etiologic agent? -Streptococcus pneumoniae -Haemophilus influenzae -Legionella +Anaerobes -Mycoplasma pneumoniae ? A 30-year-old male is admitted to the hospital after a motorcycle accident that resulted in a fracture of the right femur. The fracture is managed with traction. Three days later the patient becomes confused and tachypneic. A petechial rash is noted over the chest. Lungs are clear to auscultation. Arterial blood gases show PO2 of 50, PCO2 of 28, and pH of 7.49. Which of the following is the most likely diagnosis?
6

-Unilateral pulmonary edema -Hematoma of the chest +Fat embolism -Pulmonary embolism -Early Staphylococcus aureus pneumonia ? A 60-year-old male patient on aspirin, an angiotensin-converting enzyme inhibitor, nitrates, and a beta blocker, who is being followed for chronic stable angina, presents to the ER with a history of two or three episodes of more severe and long-lasting anginal chest pain each day over the past 3 days. His ECG and cardiac enzymes are normal. Which of the following is the best course of action? -Admit the patient and begin intravenous digoxin +Admit the patient and begin intravenous heparin -Admit the patient and give prophylactic thrombolytic therapy -Admit the patient for observation with no change in medication -Increase the doses of current medications and follow closely as an outpatient ? A 60-year-old white female presents with epigastric pain, nausea and vomiting, heart rate of 50, and pronounced first-degree AV block on ER cardiac monitor. Blood pressure is 130/80. Which of the following coronary arteries is most likely to be involved in this process? +Right coronary -Left main -Left anterior descending -Circumflex ? A 45-year-old white female smoker is admitted to the hospital for observation after presenting to the emergency department with vague chest pain. There is no past history of cardiac disease, diabetes, hypertension, or hyperlipidemia. Later that night while in bed she has a recurrence of pain, at which time cardiac monitoring shows a transient elevation of precordial ST segments. The pain is promptly relieved by sublingual nitroglycerin. Physical exam is unremarkable. Which of the following is the best follow up management plan? -Echocardiography and anti-inflammatory therapy -EGD and proton pump inhibitor therapy -Exercise stress testing; treatment depending on results +Coronary angiography; likely treatment with nitrates and calcium channel blockers -Chest CT scan; likely treatment with IV heparin ? Two weeks after hospital discharge for documented myocardial infarction, a 65-year-old returns to your office very concerned about lowgrade fever and pleuritic chest pain. There is no associated shortness of breath. Lungs are clear to auscultation and heart exam is free of significant murmurs, gallops, or rubs. ECG is unchanged from the last one in the hospital. Which of the following therapies is likely to be most effective? -Antibiotics -Anticoagulation with warfarin (Coumadin) +An anti-inflammatory agent -An increase in antianginal medication -An antianxiety agent ? This tall, thin 19-year-old white female with little previous health care complains primarily of decreased vision. You note a strong pulse, blood pressure of 180/70, and a high-pitched, blowing, diastolic decrescendo murmur? -Tricuspid stenosis
7

-Tricuspid regurgitation -Mitral stenosis -Mitral regurgitation +Aortic regurgitation (insufficiency) ? A 55-year-old type 2 diabetic patient has lost weight and has had good control of his blood sugar on oral agents. He has a history of mild hypertension and hyperlipidemia. He asks for advice about an exercise program. Which of the following statements is correct? -Exercise should be avoided because it may cause foot trauma -An active lifestyle cannot slow the complications of diabetes -Vigorous exercise cannot precipitate hypoglycemia +A stress test should be recommended prior to beginning an exercise program ? A 40-year-old cigarette smoker complains of epigastric pain, well localized, nonradiating, and described as burning. The pain is partially relieved by eating. There is no weight loss. He has not used nonsteroidal antiinflammatory agents. The pain has gradually worsened over several months. Which of the following is the most sensitive way to make a specific diagnosis? -Barium x-ray +Endoscopy -Serologic test for Helicobacter pylori -Serum gastrin ? A 36-year-old man presents for a well-patient exam. He gives a history that, over the past 20 years, he has had three episodes of abdominal pain and hematemesis, the most recent of which occurred several years ago. He was told that an ulcer was seen on a barium upper GI radiograph. You obtain a serum assay for Helicobacter pylori IgG, which is positive. What is the most effective regimen to eradicate this organism? -Omeprazole 20 mg PO daily for 6 weeks -Ranitidine 300 mg PO qhs for 6 weeks +Omeprazole 20 mg bid, amoxicillin 1000 mg bid, clarithromycin 500 mg bid for 14 days -Pepto-Bismol and metronidazole bid for 7 days ? A 73-year-old male undergoes abdominal aortic aneurysm repair. Postoperatively, his blood pressure is 110/70, abdominal aortic heart rate is 110, surgical wound is clean, and a Foley catheter is in place. His urine output drops to 40 cc/h, and creatinine rises from 1.5 to 2.2 mg/dL. Which of the following diagnostic tests is more useful for this patient? -Urine sodium +Urinalysis -Renal ultrasound -Urine uric acidurine creatinine ratio ? A 50-year-old woman complains of pain and swelling in her proximal interphalangeal joints, both wrists, and both knees. She complains of morning stiffness. She had a hysterectomy 10 years ago. Physical exam shows swelling and thickening of the PIP joints. Hemoglobin is 10.3 g/dL, MCV is 80 fL, serum iron is 8 mol/L, iron-binding capacity is 200 g/dL (normal is 250 to 370 g/dL), and saturation is 10%. Which of the following is the most likely explanation for this womans anemia? -Occult blood loss -Vitamin deficiency +Anemia of chronic disease
8

-Sideroblastic anemia ? After undergoing surgical resection for carcinoma of the stomach, a 60-year-old male develops numbness in his feet. On exam, he has lost proprioception in the lower extremities and has a wide-based gait and positive Romberg sign. A peripheral blood smear shows macrocytosis and hypersegmented polymorphonuclear leukocytes. The neurologic dysfunction is secondary to a deficiency of which vitamin? -Folic acid -Thiamine -Vitamin K +Vitamin B12 ? A 60-year-old male complains of pain in both knees coming on gradually over the past 2 years. The pain is relieved by rest and worsened by movement. There is bony enlargement of the knees with mild warmth and small effusions. Crepitation is noted on motion of the knee joint bilaterally. There are no other findings except for bony enlargement at the distal interphalangeal joint. The patient is 5 ft 9 in. tall and weighs 210 lb. Which of the following is the best way to prevent disease progression?
-Oral prednisone -Calcium supplementation -Total knee replacement -Long-term nonsteroidal anti-inflammatory drug (NSAID) administration +Weight reduction

? A 78-year-old man undergoes cardiac PTCA. Over the ensuing 2 days he develops ecchymoses and patchy digital and dermal necrosis involving his arms and legs?
-Microscopic polyangiitis -Classic polyarteritis nodosa -Giant cell arteritis +Multiple cholesterol embolization syndrome -Takayasus arteritis

? A 30-year-old paraplegic male has a long history of urinary tract infection secondary to an indwelling Foley catheter. He develops fever and hypotension requiring hospitalization, fluid therapy, and intravenous antibiotics. He improves, but over 1 week becomes increasingly short of breath and tachypneic. He develops frothy sputum, diffuse rales, and diffuse alveolar infiltrates. There is no fever, jugular venous distention, S3 gallop, or peripheral or sacral edema. Which of the following is the best approach to a definitive diagnosis in this patient?
-Blood cultures -CT scan of the chest +Pulmonary capillary wedge pressure -Ventilation-perfusion scan ? In the ICU, a patient suddenly becomes unresponsive, pulseless, and hypotensive, with cardiac monitor indicating ventricular tachycardia. The crash cart is immediately available. The first therapeutic step should be? -Amiodarone 150 mg IV push -Lidocaine 1.5 mg/kg IV push -Epinephrine 1 mg IV push +Defibrillation at 200 joules -Defibrillation at 360 joules

? A 68-year-old male with advanced CHF and BP 142/92 will have spironolactone added to his regimen, but should be informed about this possible side effect?
9

-Increased triglyceride levels -Peripheral edema -Lupus-like syndrome -Cough +Gynecomastia

? A 30-year-old male with long-standing ulcerative colitis develops jaundice, pruritus, and right upper quadrant pain. Liver biopsy shows an inflammatory obliterative process affecting intrahepatic and extrahepatic bile ducts?
-Primary biliary cirrhosis +Sclerosing cholangitis -Anaerobic liver abscess -Hepatoma -Hepatitis C

? You are helping with school sports physicals and see a 13-year-old boy who has had some trouble keeping up with his peers. He has a cardiac murmur, which you correctly diagnose as a ventricular septal defect based on which of the following auscultatory findings?
-A systolic crescendo-decrescendo murmur heard best at the upper right sternal border with radiation to the carotids; the murmur is augmented with transient exercise -A systolic murmur at the pulmonic area and a diastolic rumble along the left sternal border +A holosystolic murmur at the mid-left sternal border -A diastolic decrescendo murmur at the mid-left sternal border -A continuous murmur through systole and diastole at the upper left sternal border

? You are seeing in your office a patient with the chief complaint of relatively sudden onset of shortness of breath and weakness, but no chest pain. ECG shows nonspecific ST-T changes. You should be particularly attuned to the possibility of painless, or silent, myocardial infarction in which of the following patients?
+Elderly diabetic -Unstable angina patient on multiple medications -Premenopausal female -Inferior MI patient -MI patient with PVCs

? A 68-year-old retired construction worker has complained of aching chest pain and shortness of breath with dry cough. There is marked weight loss and anorexia. A chest x-ray shows pleural effusion with pleural thickening?
-A pH less than 7.0, 100% polymorphonuclear leukocytes -Right-sided effusion, protein 2.5 g/dL -Pleural fluid glucose less than 15 mg/dL -Exudate, 100% lymphocytes +Bloody effusion

? A 45-year-old white female has been on a diuretic, but BP remains elevated at 145/95, leading to the proposed addition of lisinopril. This key potential adverse effect should be discussed?
-Increased triglyceride levels -Peripheral edema -Lupus-like syndrome +Cough -Gynecomastia

? A 76-year old woman presents with an acute myocardial infarction.Her symptoms stared 4 hrs before admission and resolved after 1 sublingual nitroglycerine. She has history of hypertension,pulse is
10

104beats/min and BP is 180/106mmHg.A third heart sound is present.No venticular ectopy is observed druning ECG monitoring.Appropriate initial therapy may include? -Metoprolol -Digoxin +Procaineamide -Lidocaine -Diltiazem ? A 40 year old woman in good gernal health experiences sudden chest pain,fever and shortness of breath. She is heavy smoker and takes no medication except oral contraceptives.Tachypnea and temperature of 38C are found on physical examination. Chest auscultation,percussion and radiographic findings are normal.Which of the following diagnosis is likely? -Bacterial pneumonia -Atypical pneumonia -Tracheobronchitis +Pulmonary embolus -Lung cancer ? A 17y.o. patient complains of acute pain in the knee joint and T 38(C. He was ill with angina 3 weeks ago. Objectively: Deformation and swelling of the knee joints with skin hyperemia. Small movement causes and acute pain in the joints. Which diagnose is the most correct? -Systemic lupus eritematodes +Rheumatism, polyarthritis -Reactive polyarthritis -Infectious-allergic polyarthritis -Rheumarthritis ? A 12- year old boy had sever vomiting and diarrhoea.He was brought to the hosptial because of severe oliguria.He was give IV fluids and furosemide without any diuresis.Blood biochemistry revealed : Urea 120 mg\% and serum creatinine 4mg\%.The most likely diagnosis is? -Acute pyelonephritis -Haemolytic uremic syndrome -Acute glomerulonephritis -Acute tubular necrosis +Prerenal azotemia ? A 13 year old girl with sickle cell anemia presents with pallor,tachycardia,hypotension and massive splenomegaly. The most likely explanation is? -Hemorragic shock +Splenic sequestration -Septic shock -Cardiogenic shock -Hypovolemic shock ? A 30 year old female with rheumatoid arthritis of five years duration complains of pain in the frist three fingers of her right hand over past 6 weeks. The pain seems especially severe at night often awakening her from sleep. The most likelly cause is? -Carpal tunnel syndrome -Rheumatoid arthrits without complication
11

-Sensory peripheral neuropathy +Rheumatoid vacultis -Atlanto-axial sublaxation of cervical spine ? A 27 year old woman presents with muscle weakness,ptosis,has been receiving gentamicin injections for the last 7 days for a urinary infection.Thyroid function tests,serum creatine kinase, eletromyogram and musle biopsy are noraml.I/V administration of edrophoniumresults isn adramtic improvements in the pateints muscle strength. Which of the following is most likely diagnosis? +Myasthenia gravis -Toxic {drug induced}myopathy -Duchenne musclar dystrophy -Peripheral neuropathy -None of the above ? A 56 year old woman is admitted after 2 weeks of frequent vomiting. She is dehydrated. Work up shows complete gastric outlet due to longstanding peptic ulcer disease. Her serum due to longstanding peptic ulce disease. Her serum chloride is 90mEq/dl {low}, sodium 134mEq/dl [low}, bicarbonate 35mEq/dl [high}, and her urine PH is low {acidic} Which of the following is most likely diagnosis? -Splenectomy -Sigmoid colectomy -Severe Crohns disease +Segmental jejunal resection -Sigmoid colectomy ? A 47-year old man presents to his physician with progressive abdominal swelling. On examination he is found to have ascites and a tender,erlarged liver. If the patient describes the ascites as having been of abrupt onset and preceded by trauma, tha most likely diagnosis is? -Portal vein thrombosis -Congestive hear failure +Thrombosis of the hepatic vein {Budd-Chiari syndrome} -Chronic hepatitis ? A 60 year old woman was admitted in ths hospital with abdominal cramps,watery diarrhea and flushing with episodes of wheezing. Select the appropriate diagnosis? -Short gut syndrome -Gastroenteritis -Uclerative colitis -Lactose intolerance +Carcinoid syndrome ? A 20- year old woman with 3-4 months history of bloody diarrhea ,stool examination negative for a ova and parasites,stool culutre negative for Clostridium,Campylobacter and Yersinia,normal bowel series edema,hypermia and ulceration of rectum and sigmoid colon seen on sigmoidoscopic examination. Which of the following is most likely diagnosis? +Ulcerative colitis -Gastroenteritis -Carcinoid syndrome -Granulomatous colitis -Lactose intolerance ?
12

A 52 years old hard smoker patient complains of persistent cough with purulent sputum discharge especially in mornings, dyspnea provoked even by slight physical exertion, wheezing chest, tahypnoe, general weakness. He considers himself to be ill during 12 years. The overwritten conditions appear 3-4 times per year usually after common cold and have tendency to progress. About what disease do you think first of all? +Chronic obstructive lung disease -Bronchial asthma -Mucoviscidosis -Bronchoectatic disease -Aspergillosis ? Patient P. aged 62 yr was diagnosed non-hospital bilateral localized in lower lobes pneumonia group III and risk class IV with exacerbated chronic obstructive lung disease in III stage. What antibacterial preparation should be chosen for this patient for treating in the in-patient department? -Amycacin -Fluoroquinolones of the III-IV generation intravenously -Aryfromycin or quarithromycin -Clindamycin -Protected aminopenicillin intravenously or cephalosporin of the II-III generation plus macrolid ? Patient F. aged 23 yr complains of the BT elevation up to 380 C, marked itching of the skin , an appearance of rash on the whole body. She ate strawberries in the evening. What preparations should be administered first of all? -Antibiotics +Antihistamine preparations of the 3-rd generation -Histaglobulin -Autoserum -Antipyretic drugs ? Of the patient of nephrological department, who is ill on chronic glomerulonephritis, changed for the worse. The patient is delayed, it is impossible to get in touch with him, painful sensitivity is kept, integuments are pale and dry, the face is swollen, there is a fibrillary twitching of muscles, pupils are narrowed, the respiration is vesicular and weakened, the tones of heart are rhythmical, pulse beat - 96, arterial pressure - 190/115. The blood test: creatinine - 0,352 mmol/liters. What condition was developed at the patient? +Uremic coma -Hyperosmotic coma -Toxic coma -Coma, which conditioned by damage of central nervous system -Hepatic coma ? An 80-year old woman with a history of Congestive Heart Faliure develops angina pectoris.Her medications are adjusted to include furosemide,digoxin,nitroglycerine and potassium supplements.Shortky there after develops intermittent frontal throbbing headache. Which of the following should the physician do frist? -Begin propranolol -Perform a temporal artery biopsy +Discontinue nitroglycerin -Begin sublingual ergotamine -Obtain a brain CT scan ? A patient 54 years old, admitted in hospital with intensive and tremendous retrosternal pain without radiation,
13

pain lasted for 40 minutes, no changes were observed after taking nitroglycerin. Objectively : pale face, sweatting, hypotonia (pulse rate 108/min), galloprythm. ECG: elevation of ST to 6 mm at III and AVF leads. Read the above given information and select the correct diagnose from the following? +Troponin I and T -Activity of the alaninaminotrasferasa -Activity of the aspartataminotrasferasa -Activity of the general lactactdehydrogenase -Activity of the general creatinphospokenase ? A patient 60 years old with ischemic heart disease, post infract cardiosclerosis. After physical exercise, there is weakness, breathlessness at rest, strong heart beat, sputum is reddish in colour, cyanotic skin, patient is half seated, half lying , cold sweat. Breathing rate is 30/min, heart rate 100/min, blood pressure 100/60 mm.hg., different kinds of rattling, hummid sounds in lungs and heart murmurs. Which mechanism leads to development of acute heart failure? -Retention of water and sodium in the organism -Decrease of oncotic pressure in the blood plasma -Raise in activity of sympathoadrenergic system -Increase in volume of circulating blood +Increase of hydrostatic pressure in small circulating blood vessels ? A patient 28 years old, during 10 years diagnosis of reumatism, failure of aortic valve, admitted in the clinic with cardiac deficiency at the II B stage. Which of the following hemodynamics is of importance in the diagnosis of systolic dysfunction? -Dillation of the heart and decrease in heart output +Increase in minute volume of blood -Hypertrophy of the heart and decrease in heart output -Decrease in minute volume of blood -Decrease of heart output ? The patient has had the non-specific hecoid colitis for many years. He is in remission now. During dispanserisation the doctor must prescribe colonoscopy to him? -Once in 3 months -Twice a year -Each month +Each year -Once in 2 years ? Throughout the year after an attack of acute myocardial infection, a patient complains of periodic pain in the heart area. At the time of observation of the electrocardiogram except pathological Q in II, III and AVF leads other changes were not observed. Which investigation is the most apropriate in order to diagnose and treat the patient? -Coronarography -Investigation of electrolytes in blood +Hollters monitering electrocardiogram -Physical test -Stress-echocardiography ? A patient with unstable stenocardia was given a complex treatment of the following : anticoagulants, lyins nitrates, (-adrenoblockers. But on the third day of treatment the pain was continous, which tests are carried out on this patient to get the proper diagnosis?
14

-Stress-ecocardiogram +Coronarography -Test with doses of physical exercises -Oesophageal electrocardio stimulator -Myocardial syncitigraphy ? The patient who after Q positive myocardial infarction of left ventricle: experienced dyspnea, edema and hepatomegalia. Echocardiograph: Increase in end diastole volume & intraventricular pressure of left ventricle. What is the main reason of congestive heart failure of the patient? +Disease + low cardiac output + accumulation of liquid -Disease + high cardiac output + increased vascular resistance -Low cardiac out put + accumlation of Na+1 + accumulation of liquid -Increased resistance of vesseles + increased activity of sympatho adrenergic system -Increased activity of sympathoadrenegic system + increased activity of renin angiotensin aldosteron system ? A patient with hypertension in 2nd stage, has inspiratory breathelessness, cough, weakness after physical exercise. Echo cardigraphy revealed: hypertrophy of left ventricle myocardium. There is a decrease of final diastolic volume with a fraction of excreation of 58\%. Destrution of heamodynamic regulation is the main cause of failure of? -Systolic diastolic function of the heart -Systolic function of the left ventricle -Diastolic function of the right ventricle -Systolic function of the right ventricle +Diastolic function of the left ventricle ? A patient 57 years old, admitted in hospital with small focal myocardial infraction from the past 3 months is reported to have cardiac insufficiency of first stage functional class 1 (Fc1). At present he complains dyspnea, fatigue, palpitation in normal physical maintinance. Which of the following is the best method of prophylaxis for the given patient? +Treatment of the basic disease -Treatment of the cardiac insufficiency -Diet with controlled amount of liquid, salt and animal fats -Satisfactory climatic treatment -Control of physical tolerance ? A 26 years old patient was admitted to the hospital with complains of pain in the right chest, fever up to 38-40 C and cough with purulent sputum discharge. Before he has been ill within 2 weeks with common cold and lower lobe pneumonia on the right. He was treated as a out-patient but 3 days ago his general condition was worsened and above mentioned complaints was appeared. On physical examination the patient is asthenic, exhausted, with pale grey skin. An unpleasant smell from the mouth is noted. On auscultation: the air entering in the lower right lobe is diminished. In other chest there is crapitation. On percussion: there is dullness over the lower lobe on the right. CBC reveals high WBCs count (leucocytosis) with the formula band shift and low RBCs count (anemia). On the X-Ray graph the right middle and low lung fields are very dense. In the lower lobe there is intrapulmonary destructive cavity with horizontal fluid level surrounded by inflammatory wall. Haw could you define the chest disease? +Lobar pneumonia complicated by acute abscess of lung -Lung gangrene -Non complicated pneumonia -Lobar pneumonia complicated by chronic abscess of lung -Chronic lung abscess
15

? A 45-year-old woman with long-standing, well-controlled rheumatoid arthritis develops severe pain and swelling in the left elbow over 2 days. She is not sexually active. You suspect septic arthritis. If you are correct, which of the following is the most likely organism to cause septic arthritis in this case? -Streptococcus pneumoniae -Neisseria gonorrhoeae -Escherichia coli +Staphylococcus aureus ? A 50-year-old male complains of low back pain and stiffness, which becomes worse on bending and is relieved by lying down. There are no symptoms of fever, chills, weight loss, or urinary problems. He had similar pain several years ago. On exam, there is paraspinal tenderness and spasm of the lower lumbar back. There are no sensory deficits, and reflexes are normal. Which of the following is the most appropriate next step in management? -Lumbosacral spine films -Stretching exercises -Weight training +Activity as tolerated, optional 2-day bed rest -MRI ? A 22-year-old male develops the insidious onset of low back pain improved with exercise and worsened by rest. There is no history of diarrhea, conjunctivitis, urethritis, rash, or nail changes. On exam the patient has loss of mobility with respect to lumbar flexion and extension. He has a kyphotic posture. A plain film of the spine shows widening and sclerosis of the sacroiliac joints. Some calcification is noted in the anterior spinal ligament. Which of the following best characterizes this patients disease process? -He is most likely to have acute lumbosacral back strain and requires bed rest +The patient has a spondyloarthropathy, most likely ankylosing spondylitis -The patient is likely to die from pulmonary fibrosis and extrathoracic restrictive lung disease -A rheumatoid factor is likely to be positive -A colonoscopy is likely to show Crohns disease ? An elderly male presents with pain in his shoulders and hips. ESR is 105 mm/L. Which of the following is most likely diagnosis? -Ankylosing spondylitis +Polymyalgia rheumatica -Reiters syndrome -Drug-induced lupus erythematosus -Polyarteritis nodosa ? A 25-year-old male cigarette smoker has a history of respiratory infections and has also been found to have hematuria. A high value for diffusing capacity is noted during pulmonary function testing. This elevated diffusing capacity is consistent with which of the following disorders? -Anemia -Cystic fibrosis -Emphysema +Intrapulmonary hemorrhage ? A 28-year-old male with a long history of severe asthma presents to the emergency room with shortness of breath. He has previously required admission to the hospital and was once intubated for asthma. Which of the following findings on physical exam would indicate a benign course?
16

+Pulsus paradoxus of 5 mmHg -Hypercapnia -Thoracoabdominal paradox (paradoxical respiration) -Silent chest -Altered mental status ? An anxious young woman who is taking birth control pills presents to the emergency room with shortness of breath. The absence of which of the following would make the diagnosis of pulmonary embolus unlikely? -Wheezing -Pleuritic chest pain -Tachypnea +Hemoptysis -Right-sided S3 heart sound ? A 75-year-old patient presents to the ER after a sudden syncopal episode. He is again alert and in retrospect describes occasional substernal chest pressure and shortness of breath on exertion. His lungs have a few bibasilar rales, and his blood pressure is 110/80. Which of the following classic findings should you expect to hear on cardiac auscultation? +A harsh systolic crescendo-decrescendo murmur heard best at the upper right sternal border -A diastolic decrescendo murmur heard at the mid-left sternal border -A holosystolic murmur heard best at the apex -A midsystolic click ? A 72-year-old male comes to the office with intermittent symptoms of dyspnea on exertion, palpitations, and cough occasionally productive of blood. On cardiac auscultation, a low-pitched diastolic rumbling murmur is faintly heard toward the apex. The origin of the patients problem probably relates to which of the following? +Rheumatic fever as a youth -Long-standing hypertension -A silent MI within the past year -A congenital condition ? A 23-year-old graduate student comes in with extreme fatigue and a vague sense of ill feeling over the past few weeks. He states that he has been under much stress recently, and in fact is slightly agitated. On exam, BP is 110/70, pulse is 100, and temperature is 100.5F (38.0C). The neck veins are distended with prominent v waves. A holosystolic murmur is heard at the left sternal border, which intensifies on inspiration? -Tricuspid stenosis +Tricuspid regurgitation -Mitral stenosis -Mitral regurgitation -Aortic regurgitation (insufficiency) ? A 72-year-old male presents with a 2-h history of chest pain; acute ST segment elevation in leads II, III, and aVF; and sinus bradycardia at a rate of 40, with hypotension? -Amiodarone -Digoxin +Atropine -Diltiazem -Isoproterenol
17

? A 40-year-old male with long-standing alcohol abuse complains of abdominal swelling, which has been progressive over several months. He has a history of gastrointestinal bleeding. On physical exam, there are spider angiomas and palmar erythema. Abdominal collateral vessels are seen around the umbilicus. There is shifting dullness, and bulging flanks are noted. Which of the following is an important first step in the patients evaluation? +Diagnostic paracentesis -UGI series -Ethanol level -CT scan ? A 42-year-old man presents for the evaluation of splenomegaly, which was incidentally discovered on physical exam. You note mild abdominal distension with shifting dullness and perform diagnostic abdominal paracentesis. The fluid is straw-colored, nonbloody, and contains 320 white blood cells per L, 68% of which are lymphocytes. The peritoneal fluid albumin level is 1.3 g/dL; the serum albumin level is 3.8 g/dL. Which of the following is the most likely diagnosis? -Thyroid carcinoma -Pancreatitis -Tuberculous peritonitis -Hepatoma +Portal hypertension ? A 53-year-old male with septic shock develops acute renal failure with a serum creatinine of 6.4 mg/dL. Which of the following is a specific indication to initiate dialysis? -BUN rises to 75 mg/dL -Urine output falls to <10 mL/h +Pericardial friction rub develops -Hematocrit falls to <30% ? A 20-year-old male presents acutely intoxicated. Blood pressure is 120/70. His laboratory values are as follows: Na: 140 meq/L, K: 5.1 meq/L, Cl: 100 meq/L, HCO3 15 meq/L, Creatinine 1.2 mg/dL, Blood ethanol: nondetectable, Blood glucose: 110 mg/dL, Arterial blood gases: PO2 88, PCO2 28, pH 7.28 Which of the following tests will provide the key to correct diagnosis? -Serum ketones -Serum lactate -Salicylate level +Measured plasma osmolality ? A 38-year-old female presents with repeated episodes of sore throat. She is on no medications, does not use ethanol, and has no history of renal disease. Physical exam is normal. A CBC shows Hgb of 9.0 g/dL, MCV is 85 fL (normal), white blood cell count is 2,000/L, and platelet count is 30,000/L. Which of the following is the best approach to diagnosis? -Erythropoietin level -Serum B12 -Bone marrow biopsy +Liver spleen scan ? A patient complains of fatigue and night sweats associated with itching for 2 months. On physical exam, there is diffuse nontender lymphadenopathy, including small supraclavicular, epitrochlear, and scalene nodes. A chest x-ray shows hilar lymphadenopathy. Which of the following is the best next step in evaluation?
18

+Excisional lymph node biopsy -Monospot test -Toxoplasmosis IgG -Serum angiotensin converting enzyme level ? A 60-year-old male complains of shortness of breath 2 days after a cholecystectomy. There is no fever, chills, sputum production, or pleuritic chest pain. On physical exam temperature is 99F, pulse is 75, respiratory rate is 20, and blood pressure is 120/70. There are diminished breath sounds and dullness over the left base. Trachea is shifted to the left side. A chest x-ray shows a retrocardiac opacity that silhouettes the left diaphragm. Which of the following is the most likely anatomical problem in this patient?
-An acute process causing inflammation -A left lower lobe mass +Diminished lung volume in the left lower lobe, postoperatively -Acute bronchospasm caused by surgery

? A 70-year-old nonsexually active male complains of fever and pain in his left knee. Several days previously, the patient skinned his knee while working in his garage. The knee is red, warm, and swollen. An arthocentesis is performed, which shows 200,000 leukocytes/L and a glucose of 20 mg/dL. No crystals are noted. Which of the following is the most important next step?
+Gram stain and culture of joint fluid -Urethral culture -Uric acid level -Antinuclear antibod -Antineutrophil cytoplasmic antibody

? A 55-year-old woman presents with a 2-month history of fever, weight loss, dyspnea, and microscopic hematuria with RBC casts. A lung biopsy demonstrates pulmonary capillaritis. Which of the following is most likely diagnosis?
-Microscopic polyangiitis -Classic polyarteritis nodosa -Giant cell arteritis +Multiple cholesterol embolization syndrome -Takayasus arteritis

? A 20-year-old woman has developed low-grade fever, a malar rash, and arthralgias of the hands over several months. High titers of anti-DNA antibodies are noted, and complement levels are low. The patients white blood cell count is 3000/L, and platelet count is 90,000/L. The patient is on no medications and has no signs of active infection. Which of the following statements is correct?
-The patient will have a false-positive test for syphilis -Central nervous system symptoms will occur within 10 years -The patient can be expected to develop Raynauds phenomenon when exposed to cold +If glomerulonephritis, severe thrombocytopenia, or hemolytic anemia develops, high-dose glucocorticoid therapy would be indicated -The disease process described is an absolute contraindication to pregnancy

? A 40-year-old white female complains of pruritus. Physical examination reveals xanthelasma and mild splenomegaly. She has an elevated alkaline phosphatase, but her transaminases are normal. The antimitochondrial antibody test is positive. Which of the following is most likely diagnosis?
-Sclerosing cholangitis +Primary biliary cirrhosis -Anaerobic liver abscess -Hepatoma 19

-Hepatitis C

? An 18-year-old male with a history of intravenous drug abuse develops status epilepticus. His CPK rises to 30,000 IU/L, his urine output drops to 20 mL/h, and his creatinine rises to 5.6 mg/dL. Which of the following is the best initial therapy for rhabdomyolysis-induced acute renal failure?
-Mannitol -Dopamine -Natriuretic peptide +Alkaline diuresis ? A 19 years old patient complains of long-term bleeding started after tooth extraction. It is known from the case history that he has had large haematomas in the childhood. His uncle from mothers side died from bleeding. What clinical diagnosis is the most probable? +Hemophilia -Hemorrhagic vasculitis -Iron deficiency anemia -Aplastic anemia ? A man aged 32 yr notes general weakness, elevation of the body t, pains in the throat while swallowing. He is ill 2 months and treated from tonsillitis by antibiotics without any effect. The main clinical features are body t 38,4 C, RR (respiratory rate) 22 per min; pulse 108, BP (blood pressure) 100/60 mm of Hg. The skin is pale. There are purpura (hemorrhagic rash) on extremities, systemic enlargement of lymphatic nodes and hepatosplenomegaly. In CBC RBC are 2,2*1012/l, Hb 70 g/l; WBC (leuc.) 13,5*109/l; atypical cells (blastocytes) 32\%; metamyelocytes 1\%; stab neutrophils 3\%; segm. neutr. 35\%; lymph. 20\%; monocytes 8\%; thrombocytes 37*109/l; ESR 30mm/hour. What clinical diagnosis is the most probable? +Acute leukosis -Chronic lympholeukosis -Chronic myeloleukemia -Aplastic anemia -Thrombocytopenic purpura ? A men, 25 years, appealed to the policlinic concerning a quinsy. Appointed soulfanilamidi and aminoglicozidi. On 5-those day an urinary syndrome turned out, arterial hypertension was not present. A to 15 day an infection ended and an urinary syndrome gradually vanished. What diagnosis must it be? -Sharp pyelonephrit -Sharp glomerulonephrit +Infection-tocsical bud -Sheynleyns Genohs illness -Bergers illness ? The arterial hypertensive patient aged 30 during last 5 years has had a constant BP index 170/100 200/120 mm Hg. On objective examining systolic murmur is heard above the navel in the right. Whats the reason of hypertension? -hypertension disease +reno-vascular hypertension -polycystic kidney -coarctation of aorta -glomerulonephritis ? The patient aged 30 complains of headache, general weakness, nocturia. Proteinuria has been noted 5 years and arterial hypertention 2 years ago. Heart rate is 88 beats per min, blood pressure 150/100 mm Hg, creatinine 20

0.23 mkmole/l, urea 9,1 mlmole/l, glomerular filtration rate 50 ml/min. Diagnosis? -acute renal insufficiency 1 stage -chronical renal insufficiency 2 stage -Heart failure 3 stage -chronical renal insufficiency 4 stage +chronical renal insufficiency 1 stage ? The patient aged 31, called in with complaints of headache, nausea, vomiting. Diagnosis of chronical glomerulonephritis with renal failure was made in stationary. What medical drug again high blood urea nitrogen would you recommend to the patient? +Polysorb -Furosemid -Hypothiazid -Guttalax -Verapamil ? A sick man, who is 43, started to complain to his therapeutist about the heaviness in the stomach after a meal, bad belch, vomiting with food, he had eaten the day before. He has suffered from ulcer of duodenum for 10 years. He has applied to the doctor (therapeutist) two times but he started to feel worse last month. What pathology must it be? -Gullet cancer +Ulcerous stenosis of the exit part of the gullet -Stomach cancer -Ahalaziya of the gullet -Reactive pancreatitis ? The patient, aged 65 rising to the first floor have felt pain behind the sternum lasting nearly 5 8 minutes radiated to the left shoulder and blade. For the next 10 days the attacks of pain have become more frequent. A feeling as if he lacks air to breathe (breathleeness) was appeared. The pain didnt cease after taking one nitroglycerin tablet. What is the most probable diagnosis? -Myocardial infarction -Prinzmetals angina -Angina on exertion III funct.class +Progressive stenocardia -Acute myocarditis ? Patient after over cooling complained about continuos fever up to 38?C, cough with small amount of serous sputum. Was under treatment at district policlinic, in ten days hectic fever appeared in spite of treatment. In three days after this suddenly expectorated sputum with full mouth and very bad smell. Amphoric respiration was auscultated over middle part of the upper lobe from the right side. About what disease is it necessary to think? +Pulmonary abscess -Bronchiectatic illnes -Lobar pneumonia -Pulmonary tuberculosis -Cancer of a lung ? The patient aged 25 yr has dyspnea, cyanotic skin pallor occurred suddenly. Cardiac rate is 130 beats per minute. BP is 85/50 mm. Hg. The heart sounds are significantly quiet. On ECG there are polytopic premature bites, diffuse myocardial changes. On X-ray there is a cardiomegaly. The onset of illness the patient connects with analgin intake adverse reaction occurred 15 days ago. What is the most probable diagnosis? -Exudative pericarditis -Hypertrophic cardiomyopathy +Abramov Fiedlers idiopathic myocarditis 21

-Endocardial fibroelastosis -Mitral stenosis ? A patient aged 26 years complains of the enlargement of cervical, subclavicular, grain lymphatic nodes, skin itching, considerable sweating at night, fever up to 390C. In CBC there are leukocytosis, lymphopenia, high ESR. What clinical diagnosis is the most probable? -infectious mononucleosis -chronic lymphatic leukemia -acute lymphoblastic leukosis +lymphogranulomatosis -leukomoidal reaction ? A 42 years old man suddenly felt worsening of his general condition and complains of dizziness, weakness and appetite loss. From the clinical history it is known that the patient suffers from the duodenal ulcer disease within 15 years and it is written down in his individual medical file. In CBC erythrocytes(RBC) are 1,9*1012/l , Hb 57g/l , KP 0,9, reticulocytes 1\% , thromb. 2,1*109/l , WBC (leuc.) 11,5*109/l , eosinophils-1\%; stabneutrophils-10\%; seg.neutrophils-65\%; lymph/-19\%; mon.-5\%; ESR-35 mm/hour. What clinical diagnosis is the most probable? +post-hemorrhagic anemia -hemolytic anemia -hypoplastic anemia -DVC syndrome -pernicious anemia ? The therapeutist must prescribe to patients primary prophylaxis iron deficiency anemia to prevent its development. Wh is adminested this prophylaxis? -Patients after 60 +Pregnant women -All children -Patients after operation -Workers of industrial interprises ? The man os 25 years complains of periodic epigastralgia. At inspection the chronic gastritis with the increased acidity is revealed. Treatment is lead. What preparation is expedient for using for primary preventive maintenance of a stomach ulcer? -Maalox -Cerucal -Vicalinum +Famotidin -Gastropharm ? A 21 years old patient with history of multiple pneumonias complains of malaise, breathlessness, palpitation, fillings of noisy ears and flies in front of eyes. The patient is ill within one month. On physical examination the skin is pale. In the mouth there is ulceretive necrotizing stomatitis. Also purpura in the femoral and abdominal skin areas is noted. The peripheral lymphatic nodes are not enlarged. CBC: RBC are 1,8*1012/l, Hb 56g/l K.P0,9 , WBC 2,3*109/l; eosinohyils-2\% , stub neutrophils 3\%, segmented neutrophils 65\%, lymphocyes 22\%, monocytes 8\%, thrombocytes 16*109/l , ESR 25mm/h. In urine analysis the macrohaematuria is revealed. What clinical diagnosis is the most probable? -acquired hypoplastic anemia -hemolytic anemia -iron-deficiency anemia -B12-dificiency anemia +aplastic anemia, hemorrhagic syndrome 22

? A 70 year old patient has got an unhospital pneumonia of the second category. Prescribe the empiric therapy that is most appropriate for this case? -Ampicillin 0,5 6 times a day -Penicillin intramusculary +Spiramycin in combination with doxycycline per os -Etazol endovenous -None of the variants is right ? Patient P., age 52 has IHD (ischemic heart disease) for 5 years. He is worried about the retrosternal pain that occurs after physical exerxises. The pain lasts for 7-8 minutes and stops indepently at rest. The patient doesnt use nitrats now. There are no pathologic changes of segment ST and wave T on the ECG at rest; during the coronary angiography, decrease in diameter of the left coronary arteria by 50\% was found. What is the pathogenesis of retrosternal pain development of the patient in this case? +The incapacity of injuried coronary arteria to increase its opening on when oxygen in needed -The presence of the spasm in injured coronary arteria -Pathological adhession and aggregation of the thrombocytes with the extraction of vasoactive substanses -Hemodynamic factors of blood flow in the time of efforts in aorta -The breach of the coronary blood flow on account of arteritis ? The atrial flutter with heart rate up to 130 per min firstly appeared in a woman aged 28 yr considered before healthy. Physical examination reveals pale bluish cheeks. The respiratory rate is 24 per min. Cardiac auscultation is difficult because of high rate but S1 tone is loud. The chest is free and abdomen is soft and painless. ECG revealed right ventricle hypertrophy and atrial flutter. This woman was not consulted by any doctor because she lives in a small village. What is the most probable diagnosis? -Myocarditis +Mitral stenosis -Thyrotoxicosis -Artherosclerotic cardiosclerosis -Myocardiodistrophy ? In the patient, aged 45 after physical exertion burning pain near the heart, fear of death, sweating occurred. On the ECG there are the QS wave, cupola like ST elevation and negative T in I, AVL, V1 V3 leads. What is the most probable diagnosis? +Acute myocardial infarction of the anterior wall of left ventricle -Progressive stenocardia -Acute myocarditis -Lung artery thromboembolism -Aortic dissection ? Patient A. Complains about the shortness of breath, pains in area of heart, sense of widespread pulsation. In anamnesis - frequent quinsies (tonsillitis). Objectively: skin covers are pale; positive capillary pulse, dance of carotids, dome-shaped apex beat displaced downward and to the left, a pulse is quick, high. The left border of heart is displaced to the left. Weakening I and II tones, diastolic murmur over aorta. BP - 180/40 mm Hg. ECG: signs of hypertrophy of left ventricle. What most credible preliminary diagnosis? -Chronic nephritis -Feochromocytoma +Insufficiency of aortic valve -Hypertensive illness -Coarctation of aorta ? Patient 47 years is delivered at a clinic due to the attack of difficulty in breathing. Objectively: skin is pallidcyanotic. Position abed semisitting. Breathing is bubbling, 36 per a minute. The heart sounds are dull, accent of II 23

tone above an aorta. Above lungs different moist rales are hearing to. The WBC10,5G/l; ESR 4 mm/hour; AST 1,8 mkmol/l; ALT 1,2 mkmol/l; KFK 2,8 mkmol/l. Set a diagnosis? -Acute pleuropneumonia -Attack of bronchial asthma -Acute bronchopneumonia +Acute myocardial infarction -Pneumothorax ? Patient 28 years complains about the suddenly appearing strong and frequent blows of heart. In anamnesis the cardiac diseases are not present, was much nervous. Objectively: a patient is excited, pulse 160 in a minute, BP150/30 of mm Hg, contractions of heart are rhythmic, sounds loud, clear. ECG: heartbeats 160 per a minute, the QRS complex is stored, the P wave is not determined. In the chest leads the voltage of the T wave is megascopic. Your diagnosis? -Atrioventricular block of the III digree -Atrial flutter -Atrial fibrillation +Paroxysmal tachycardia -Respiratory arrhysmia ? Patient 50 years, after heavy physical loading substernal pain of cutting character appears suddenly. BP-140/80 of mm Hg; pulse-80 in a minute. The rhythm of heart regular, heart sounds clear. Pain disappeared in 10 minutes independently. It is known from anamnesis, that except attack of ARVI a patient was nothing ill. About what is it necessary to think? -Acute pericarditis -Diaphragmal hernia -Prinzmetals angina pectoris -Myocardial infarction +First arising angina pectoris up ? At a patient 58 years, 2 years being ill of stable angina pectoris of tension, the character of clinical displays of angina pectoris changed suddenly, more often and making of attacks heavier appeared, began anymore to accept nitroglycerine, an effect from acceptance diminished. About what is it necessary to think? +Progressive angina pectoris -First arising angina pectoris up -Stable angina pectoris of tenssion -Myocardial infarction -Acute myocarditis ? A patient 56 years is at a clinic concerning the acute myocardial infarction the third week. He complains about the increase of temperature, pains in area of heart of a different intensity and character, pains in area of humeral joint and defeat of skin. There is leukocytosis in a blood, eosinophiliya, the ESR, level of (-2 and (-globulin, C-reactive protein increased. About what is it necessary to think? -Aneurysm of heart -Heart postinfarction cardiac insufficiency -Tromboemboliya of pulmonary artery +Heart postinfarction syndrome of Dressler -Pleurisy ? A patient 65 years during a few days disturbs pain in an epigastric region, once or twice was vomiting. Appealed to the doctor. It was diagnosed acute gastritis. Got medical treatment. However much being of patient continued to get worse weakness, pallor of skin covers, sticky sweat, BP-80/50 mm Hg, heart sounds very dull. About what it is necessary to think? 24

+Diaphragmal myocardial infarction -Peptic ulcer -Acute gastritis -Solyarit -Diaphragmal hernia ? Patient 30 years with complaints about the expressed shortness of breath and palpitation during the last year. On Xray of the chest considerable expansion of heart is exposed. On an electrocardiogram decline of amplitude of the R wave in all leads and negative T. On echocardiogram dilatation of ventricles of heart with the decline of fraction of the ejection to 35 \%. A valvular apparatus not changed. It is possible to suppose? -Insufficiency of mitral valve -Insufficiency of aortic valve +Dilated (congestive) cardiomyopathy -Myocarditis -Innate heart disease ? Woman, 32 years, during 2 years was treated by internist because of neurocyrculatory dystonya and by gynecologist because of menorrhagia. Complaints about the promoted fatigability, muscular weakness, predilection for strong smells (petrol, ether), the desires eat chalk. Common analysis of blood: RBC- 3,5 T/l; Hb 105 g/l; C.i. 0,8; hypochromiya, anisocytosis; WBC. - 6,8 g/l; B-0\%; E-2\%; St-4\%; S-60\%; L-26\%; M-8\%; ESR 12mm/ch. Whey iron 7-8 mkm/l. What is it necessary to appoint? +Peroral preparations of iron -Vitamins of B group -Folic acid -Parenteral preparationsof iron -Vitamin C ? A 45-year-old woman has pain in her fingers on exposure to cold, arthralgias, and difficulty swallowing solid food. Of the following tests, which, if positive, would be most supportive of a definitive diagnosis? -Rheumatoid factor +Antinucleolar antibody -ECG -BUN and creatinine -Reproduction of symptoms and findings by immersion of hands in cold water ? A 20-year-old male complains of arthritis and eye irritation. He has a history of burning on urination. On exam, there is a joint effusion of the right knee and a dermatitis of the glans penis. Which of the following is correct? -Neisseria gonorrhoeae is likely to be cultured from the glans penis -The patient is likely to be rheumatoid factorpositive +An infectious process of the GI tract may precipitate this disease -An ANA is very likely to be positive ? A 43-year-old man with diabetes and cardiomegaly has had an attack of pseudogout. He should be evaluated for which of the following? -Renal disease +Hemochromatosis -Peptic ulcer disease -Lyme disease -Inflammatory bowel disease ? A 78-year-old man presents with a 2-month history of fever and intermittent abdominal pain. He develops peritoneal signs and at laparotomy is found to have an area of infarcted bowel. Biopsy shows inflammation of small 25

to medium-sized muscular arteries. Which of the following is most likely diagnosis? -Microscopic polyangiitis -Takayasus arteritis -Giant cell arteritis -Multiple cholesterol embolization syndrome +Classic polyarteritis nodosa ? A 65-year-old male with mild congestive heart failure is to receive total hip replacement. He has no other underlying diseases and no history of hypertension, recent surgery, or bleeding disorder. Which of the following is the best approach to prevention of pulmonary embolus in this patient? -Aspirin 75 mg/d -Aspirin 325 mg/d +Warfarin with INR of 2 to 3 or low-molecular-weight heparin -Early ambulation ? A 30-year-old athlete with asthma is also a cigarette smoker. Which of the following is characteristic of asthma but not other obstructive lung disease? -Hyperinflation is present on chest x-ray +Airway obstruction is reversible -Hypoxia occurs as a consequence of ventilation-perfusion mismatch -The FEV1/FVC ratio is reduced -Exacerbation often occurs as a result of an upper respiratory tract infection ? A 50-year-old male with emphysema and a chest x-ray that has shown apical blebs develops the sudden onset of shortness of breath and left-sided pleuritic chest pain. Pneumothorax is suspected. Which of the following physical examination findings would confirm the diagnosis? -Localized wheezes at the left base -Dry crackles at both bases -Increased tactile fremitus on the left side -Decreased breath sounds on the left side with deviation of the trachea to the left +Hyperresonance of the left chest with decreased breath sounds ? A 35-year-old female complains of slowly progressive dyspnea. Her history is otherwise unremarkable, and there is no cough, sputum production, pleuritic chest pain, or thrombophlebitis. She has taken appetite suppressants at different times. On physical exam, there is jugular venous distention, a palpable right ventricular lift, and a loud P2 heart sound. Chest x-ray shows clear lung fields. ECG shows right axis deviation. A perfusion lung scan is normal, with no segmental deficits. Which of the following is the most likely diagnosis? +Primary pulmonary hypertension -Recurrent pulmonary emboli -Cardiac shunt -Interstitial lung disease ? A 40-year-old male in generally good health presents to the office with a history of palpitations that last for a few seconds and occur two or three times a week. There are no other symptoms. ECG shows a rare single unifocal premature ventricular contraction (PVC). Which of the following is the most likely cause of this finding? -Underlying coronary artery disease -Valvular heart disease -Hypertension -Apathetic hyperthyroidism +Idiopathic or unknown ? 26

A 30-year-old female presents with a chief complaint of palpitations. A 24-h Holter monitor is obtained and shows occasional unifocal PVCs and premature atrial contractions. Which of the following is the best antiarrhythmic management in this patient? -Anxiolytic therapy -Beta blocker therapy -Digoxin +Reassurance, no medication -Quinidine ? An active 78-year-old female has been followed for hypertension but presents with new onset of mild left hemiparesis and the finding of atrial fibrillation on ECG, which persists throughout the hospital stay. She had been in sinus rhythm on checkup 3 months earlier. Optimal management by the time of hospital discharge includes review of antihypertensive therapy, a ventricular rate control agent if needed, plus which of the following? -Automated implanted cardioverter-defibrillator (AICD)/permanent pacemaker placement to avoid the need for anticoagulation -Waiting for anticoagulation therapy until the ability to ambulate without falls is established -Antiplatelet therapy such as aspirin, without warfarin (Coumadin) -Warfarin (Coumadin) with a target INR of 1.5 plus antiplatelet therapy +Warfarin (Coumadin) with a target INR of 2.5 ? A 55-year-old African American female presents to the ER with lethargy and blood pressure of 250/150. Her family members indicate that she was complaining of severe headache and visual disturbance earlier in the day. They report a past history of asthma but no known kidney disease. On physical exam, retinal hemorrhages are present. Which of the following is the best approach? -Intravenous labetalol therapy +Continuous-infusion nitroprusside -Clonidine by mouth to lower blood pressure slowly but surely -Nifedipine sublingually to lower blood pressure rapidly -Further history about recent home antihypertensives before deciding current therapy ? An 18-year-old male is sent over from his physical education class due to his symptoms of dizziness and palpitations after exercise. The instructor thinks he may be faking this to get out of future activities. Vital signs are within normal limits. A rapidly rising carotid pulse is noted. On auscultation an S4 is heard along with a harsh systolic crescendodecrescendo murmur, beginning well after S1, best noted at the lower left sternal border. Which

of the following is most likely diagnosis?


-Tricuspid stenosis -Tricuspid regurgitation +Hypertrophic cardiomyopathy -Mitral regurgitation -Aortic regurgitation (insufficiency) ? A 40-year-old alcoholic male is being treated for tuberculosis, but he has not been compliant with his medications. He complains of increasing weakness and fatigue. He appears to have lost weight, and his blood pressure is 80/50 mmHg. There is increased pigmentation over the elbows. Cardiac exam is normal. Which of the following is the best next step in evaluation? -CBC with iron and iron-binding capacity -Erythrocyte sedimentation rate +Early morning serum cortisol and cosyntropin stimulation -Blood cultures ?

27

A nursing student has just completed her hepatitis B vaccine series. On reviewing her laboratory studies (assuming she has no prior exposure to hepatitis B), you should expect which of the following? -Positive test for hepatitis B surface antigen +Antibody against hepatitis B surface antigen (anti-HBs) alone -Antibody against hepatitis core antigen (anti-HBc) -Antibody against both surface and core antigen -Antibody against hepatitis E antigen ? A 40-year-old male has a history of three duodenal ulcers with prompt recurrence. Symptoms have been associated with severe diarrhea. One of the ulcers occurred close to the jejunum. Serum gastrin levels have been above 400 pg/mL (normal is 40 to 200 pg/mL). Which of the following is the most useful test in this patient? -Colonoscopy -Endoscopic retrograde cholangiogram -CT scan of abdomen +Secretin injection test ? A 70-year-old male is found lethargic at home with a blood pressure of 98/60 and a temperature of 98.6F. In the emergency room, the following laboratory studies are obtained:Na: 138 meq/L, K: 2.8 meq/L, HCO3: 10 meq/L, Cl: 117 meq/L, BUN: 20 mg/dL, Creatinine: 1.0 mg/dL, Arterial blood gases: PO2 80, PCO2 25, pH 7.29, Urine pH: 4.5. Which of the following is the most likely acid-base disorder? +Non-anion-gap metabolic acidosis -Respiratory acidosis -Anion-gap metabolic acidosis -Non-anion-gap metabolic acidosis and respiratory alkalosis ? A 64-year-old male is hospitalized with a transient ischemic attack and is evaluated for carotid disease. Physical exam is normal. CBC on admission is normal. The patient is started on heparin. A repeat CBC 1 week later shows Hgb of 14 g/dL (normal is 13 to 18 g/dL), WBC of 9000/mL, and platelet count of 10,000/mL. Which of the following is the most appropriate course of action? -Obtain a bone marrow study -Obtain a liver-spleen scan +Suspect drug-induced thrombocytopenia -Begin corticosteroids for idiopathic thrombocytopenia purpura ? A young male presents with leg swelling and recurrent aphthous ulcers of his lips and tongue. He has also recently noted painful genital ulcers. There is no urethritis or conjunctivitis. On exam, he has evidence of deep vein thrombophlebitis. Which of the following is most likely diagnosis? -Drug-induced lupus erythematosus -Ankylosing spondylitis -Polymyalgia rheumatica -Reiters syndrome +Behets syndrome ? An ECG is brought to you with notation of this being a 62-year-old male with small cell carcinoma of the lung and hyponatremia. The electrocardiographic finding most likely to occur in this case is which of the following? +No abnormal change -Shortened PR interval -Prolonged PR interval -Convex elevation of the J point (Osborn wave) -Diffuse ST-segment elevation ? 28

A 72-year-old male presents to the ER with the chief complaint of shortness of breath that awakens him at night. Physical exam findings include bilateral basilar rales, an S3 gallop, and evidence of cardiomegaly, all major criteria for the diagnosis of congestive heart failure under the Framingham system. While patients with CHF may display a wide range of symptoms and signs, which of the following are also considered among the major criteria (i.e., more specific to CHF)? -Dyspnea on exertion and night cough -Right-sided pleural effusion plus hepatomegaly -Tachycardia (rate 120 or greater) with new onset or increase in ectopy +Neck vein distention and positive hepatojugular reflux -Extremity edema and weight gain greater than 2 kg over 1 week ? A 78-year-old white man presents with a 3-day history of gradually worsening left lower quadrant pain. He denies rectal bleeding or weight loss but has noticed mild constipation in association with the pain. He has a temperature of 100.2 F, moderate left lower quadrant tenderness without evidence of peritoneal inflammation, and a white count of 14,200. Which of the following is most likely diagnosis? +Acute diverticulitis -Acute pancreatitis -Acute cholecystitis -Intestinal obstruction -Irritable bowel syndrome ? A 17-year-old male is brought to the emergency room with confusion and incoordination. Laboratory values are as follows: Na: 135 meq/L, K: 2.7 meq/L, HCO3: 15 meq/L, Cl: 110 meq/L, Arterial blood gases: PO2 92, PCO2 25, pH 7.25, Urine: pH 7.5, glucosenegative, Ca: 9.7 mg/Dl, PO4: 4.0 mg/dL Which of the following is the most likely cause of the acid base disorder? -GI loss due to diarrhea -Proximal renal tubular acidosis -Disorder of the renin-angiotensin system +Distal renal tubular acidosis ? A 32-year-old female is referred to you from an OB-GYN colleague due to the onset of extreme fatigue and dyspnea on exertion 1 month after her second vaginal delivery. By history, physical exam, and echocardiogram, which shows systolic dysfunction, you make the diagnosis of peripartum (postpartum) cardiomyopathy. Which of the following statements is correct? -Postpartum cardiomyopathy may occur unexpectedly years after pregnancy and delivery +About half of all such patients will recover completely -The condition is idiosyncratic; the risk of recurrence in a future pregnancy is no greater than average -The postpartum state will require a different therapeutic approach than typical dilated cardiomyopathies ? A 64-year-old woman is found to have a left-sided pleural effusion on chest x-ray. Analysis of the pleural fluid reveals a ratio of concentration of total protein in pleural fluid to serum of 0.38, a lactate dehydrogenase (LDH) level of 125 IU, and a ratio of LDH concentration in pleural fluid to serum of 0.46. Which of the following disorders is most likely in this patient? -Bronchogenic carcinoma -Systemic lupus erythematosus -Pulmonary embolism -Sarcoidosis +Congestive heart failure ? The woman aged 30, is suffering from polycystic kidney. She was admitted to the hospital because of weakness thirst, nocturia. Diuresis was about 1800 ml per day. Blood pressure 200/100mm Hg. Blood test: erythrocytes: 1,8*109. Hg-68 g/l. Urine specific gravity is 1005, leucocytes- 50-60, erythrocytes in microscopic fild - 3-5, 29

creatinine- 0,82 mmole/l, potassium 6,5 mmole/l, provision urine filtration rate 10 ml/min. Whats the immediate task in the case treatment? -blood transfusion -antibacterial therapy -sorbents +Hemodialysis -Hypotensive therapy ? A patient is 60 years. In anamnesis - hypertensive illness, angina pectoris. Felt off suddenly palpitation, heaviness in area of heart, difficult breath appeared. At examination: it is pale, it is all of a cold sweat, lips are cyanotic. A pulse is frequent, it is difficult to count up. BP 100/50 mm Hg. The heart sounds are dull. A patient is quickly hospitalized. How do you think, with what diagnosis? +Myocardial infarction -Hypertensive crisis -Attack of angina pectoris -Fainting-fit -Cardiac asthma ? The sick man aged 40 has been suffering from glomerulonephritis. He complains of vomiting, muscle cramps in the distal extremity. Blood pressure 180/120 mm Hg, creatinine in blood serum 770 mkmole/l, glomerular filtration 5 ml/min. What treatment methods are indicated to this patient ? -hemo absorption -sorbents +hemodialisis -plasmapheresis -hemofiltration ? A patient 55 years complains about pain, slight swelling in the joints of brushes, long constraint at mornings, limitation of mobility. 1 year is ill. Treated oneself with ibuprofen with a small effect. Objectively: swelling of metacarpal-phalange, proximal interphalange joints of the II-III fingers of both brushes with pain limitation of mobility. ESR 37 mm/hour. What researches are most informing for clarification of diagnosis of rheumatoid arthritis? -Presence of LE-cells -Determination of uric acid in blood -Titers of antichlamidias antibodies +Rheumatoid factor, X-ray of brushes joints -Determination of blood lipids level ? A patient 24 years complains about nausea, vomiting, headache, shortness of breath. In 10 years had an acute nephritis. Proteinuria was found out in urine. Objectively: a skin is grey- pale, the edema is not present. Accent of II tone above an aorta. BP 140/100 - 180/100 mm Hg. Blood level of residual N2 6,6 mmol/L, creatinine 406 mol/L. Day's diuresis 2300 ml, nicturia. Spac. gravity. of urine is1009, albumen 0,9 g/L, WBC-0-2 in f/vis. RBC.-sin. in f/vis., hyaline casts single in specimen. Your diagnosis? +Chronic nephritis with violation of kidney function -Feochromocitoma -Hypertensive illness of the II degree -Nephrotic syndrome -Stenosis of kidney artery ? A patient 70 years suffers of hypertensive illness more than 20 years. Woke up at night due a suddenly arised attack of shortness of breath with difficult inhalation. Objectively: orthopnoe. Pulse 108 per/min, rhythmic. The heart sounds are dull, on an apex the rhythm of gallop. In the lower parts of lungs moist wheezes. What complication of hypertensive illness arose up at a patient? 30

-Acute violation of cerebral blood circulation -Myocardial infarction +Cardiac asthma -Fainting-fit -Pulmonary edema ? Patient M., 52 years. Complains about headache, palpitation. About 5 years suffers of hypertension. Had the myocardial infarction one year ago, 6 months ago - cerebral stroke. Objectively: being of middleweight. The left border of heart is displaced laterally on 2,5 sm. Accent of II tone on an aorta, systole murmur on an apex. Ps - 96 in a minute, arrhythmic. BP 190/105 mm Hg. Your diagnosis? -Coarctation of aorta -Stenosis of bulb of aorta -Insufficiency of aortic valve +Hypertensive illness of the III degree -Atherosclerosis of aorta ? Man 40 years complains about gradually increasing headache, nausea, inclinations to vomiting, somnolence, pains in a heart, fog before eyes. Suffers with hypertensive illness 12 years. Closing dates much worked, got tired. Objectively: it is pale, a face is puffy, Ps-62 /min, is hard. BP 200/120 mm Hg. The left border of heart is displaced to the left on 2 cm. Accent of II tone above an aorta, slight edema of shins. Specific gravity of the urine is 1015. WBC - single in f/vision. ECG: signs of hypertrophy of left ventricle. Your diagnosis? +Hypertensive illness of the II degree, crisis -Ishemic heart disease -Chronic pyelonephritis -Itsenko-Kushing Illness -Coarctation of aorta ? Patient I., 50 years, complains about headache, flashing of beauty spot before eyes, excitation, trembling in all body. Suffers of hypertensive illness 5 years. Arterial pressure 140/90 - 180/100 mm Hg. Objectively: skin covers are moist, red color. Ps is tense, 96 per/min. BP - 190/105 mm Hg. Left border of cardiac dullness - 1 cm laterally from a left middle clavicular line. Accent of II tone on an aorta. ECG has the signs of hypertrophy of the left ventricle. Spec. grav. of urine 1019. WBC. - 2-4 in f/vis. RBC. - 0-1 in f/vis. Your diagnosis? -Acute glomerulonephritis -Feochromocitoma +Hypertensive illness of the II degree, crisis -Atherosclerosis of aorta -Insufficiency of aortic valve ? A patient has complaints about a tormental (agonizing) cough with expectoration to 600-ml/daily purulent chocolate color of sputum with a decay smell. Onset of illness was abrupt, temperature 39(C, fever of irregular type. There is the area of darkening with a cavity in a center on X-ray film, with irregular contours and level of liquid. What disease is the question? +Gangrene of lung -Tuberculosis -Bronchiectatic illness -Abcetic pneumonia -Lobar pneumonia ? A female patient complains of weakness, dizziness, breathlessness on physical exertion and, diarrhea. In history the patient started to be ill 3 years ago after operative stomach resection by B-II done against a chronic gastric ulcer disease complicated with severe bleeding. On physical examination the skin is pale and tongue brightly lacquered red. The CBC reveals a low HB and hyperchromic enlarged in size RBCs, their degenerative forms and poikilocytosis, Zholys intra cell inclusions. She was treated by iron preparations and vitamin B12 and then her condition became much better. What is the type of anemia? 31

-Coexistence of Iron deficient anemia with hereditary 12 folic acid deficient anemia +Anemia due to gastricectomy -Iron deficient anemia -12 folic acid deficient anemia -Aplastic anemia ? Disease began abruptly with shaking chills, severe headache and fever up to 39-40C. Pain appears from the left side of the chest in 4 hours during respiration. Cough at once was dry, but in 4 days rusty sputum was expectorated. Percussion revealed hip sound over left low lobe, vocal fremitus was intensified, during auscultation was listened bronchial respiration over affected lobe. What percussion sound will be over space of Traube? -Clear lung sound -Bandbox sound +Tympanic sound -Dull-tympanic -Hip sound ? Woman, 56 years, complaints about the skin itching, sweating, subfebrile fever with the periodic increases of temperature to 38-39 C(, loss of weight, heaviness in right and left hypochondria, dyspeptic disorders. Objectively: a skin is pale, some icteric. A liver palpate at the level of umbilicus, a spleen on 2 cm is below than umbilicus, dense consistency. Common analysis of blood: RBC -3,0 T/L, Hb-110 g/L, C.I.-0,8; WBC-28 G/L, bas. 4\%, eos.5, myel.-6, St.-8, seg.-70, L-5, M-3, trom.180g/L,ESR mm/hour. Your diagnosis? +Chronic myeloleucosis (subleucemic form) -Chronic myeloleucosis (leucemic form) -Chronic myeloleucosis (Aleucemic form) -Acute myeloblastic leucosis -Chronic lympholeucosis ? Rectoromanoscopy showed a 1 cm polyp in the patient. On histological examination there was found an adenomatous polyp. The further reasonable step in treating this patient can be? -repeated rectoromanoscopy -rectum resection -endoscopy polypectomy -repeated analysis of the occult blood in feces +irrigoscopy ? The patient, 45 years, show complaints on severe retrosternal pains, not decreasing after usage of nitroglycerine. Objectively: integuments are pale, there is cold perspiration, the respiration is vesicular and weakened, the tones of heart are rhythmical, pulse beat - 78, arterial pressure - 110/70. On the electrocardiogram: a rhythm is right sinus, there is pathological wave Q in the V1-V4, depression of segment ST in III AVF. What is your preliminary diagnosis? -Thromboembolism of the pulmonary artery -Attack of a stenocardia +Acute miocardial infarction anterior-septal-apex -Acute miocardial infarction of posterior wall of the left ventricular -Acute pericarditis ? Of the patient of nephrological department, who is ill on chronic glomerulonephritis, changed for the worse. The patient is delayed, it is impossible to get in touch with him, painful sensitivity is kept, integuments are pale and dry, the face is swollen, there is a fibrillary twitching of muscles, pupils are narrowed, the respiration is vesicular and weakened, the tones of heart are rhythmical, pulse beat - 96, arterial pressure - 190/115. The blood test: creatinine - 0,352 mmol/liters. What condition was developed at the patient? +Uremic coma 32

-Hyperosmotic coma -Toxic coma -Coma, which conditioned by damage of central nervous system -Hepatic coma ? Patient C aged 47 yr complains of intensive skin itching, jaundice, bone pain. The skin is hyperpigmentated. There is multiple xanthelasma palpebrarum. The liver is +6 cm enlarged, hard with acute edge. The blood analysis revealed total bilirubin 160 mkmol/l, direct 110 mkmol/l, AST (asparate aminotransferase) 2,1 mmol/l per hour, ALT 1.8 mmol/l, alkaline phosphotase 4,6 mmol/l per hour, cholesterol 9,2 mmol/l, antimitochondrial antibodies M2 in a high titer. What is the probable diagnosis? -primary liver cancer +primary biliary liver cirrhosis -chronic viral hepatitis B -acute viral hepatitis B -alcoholic liver cirrhosis ? The patient aged 60 yr feels dyspnea on a even slight physical exertion and complains of lower limbs swelling in the daytime. He is suffering from Ischemic Heart Disease and persistent atrial flutter within 2 years. On physical examination the patient appears acrocyanotic. Heart rate is 120 per min., PS 68, BP 105/63 mm Hg. Heart sounds are loud, arrhythmic. Chest auscultation reveals moist rales in the lower lung regions. What treatment is the most advisable for this patient? -Corinfar -Egilok -Isoptine +Digoxin -Preductal ? The complications of acute cholecystitis which require surgical intervention are as follows except? +Jaundice -Empyema of the gall-bladder -Emphysematous gall-bladder -Gall-bladder perforation -Cholangitis conditioned by the presence of stones in the bile tract ? The serologic profile of the patient having been inoculated with recombinant vaccine against hepatitis B (HBV includes the positive result of the test for? -anti-HBe -HBsAg +anti- -anti--HBs -anti - HBc and anti -HBs ? Which of the following substances (drugs) can cause granulomatous hepatitis? -methyldopha -alchohol -paracetamol +allopurinolum -cordaron ? The patient is 36. For a few years he has complained of heaviness and a dull ache in the abdomen. The liver is enlarged, hard and has an uneven surface. CT (computer tomography) has shown multiple pathological changes in the organs of abdominal cavity. The correct diagnosis is? 33

-acute hepatitis -cirrhosis of the liver -multiple metastases of liver cancer -echinococcosis of the liver +polycystosis of the liver ? In the patient with the pulsating formation in the epigastrium where systolic murmur has been heard a sudden acute pain in the abdomen has occurred. After the shock condition with loss of consciousness was developed. The diagnosis is? +aneurysm rupture -ulcer perforation -acute appendicitis -peritonitis -acute pancreatitis ? The 48 years old patient has the left ventricle wall hypertrophy according to the data of echocardiography. What disease has no such a symptom ? -Hypertonic disease -Ischemic heart disease +Dilatative cardiomyopathy -Hypertrophic cardiomyopathy -Myocarditis ? Splenomegaly, the low RBC (red blood cells) count, the high content of urobilinogen in feces of the patient with jaundice are the symptoms of ? -extrahepatic cholestasia +hemolytic jaundice -intrahepatic cholestasia -sepsis -mechanical jaundise ? The patient, 32 years, complains of a constant aching lumbar pains, frequent painful urination, subfibrile temperature, a headache. Urine test: relative density - 1015, protein - 0,066 gr/liters, leukocytes on all field of vision, erythrocytes - 4-5, oxalates. On the roentgenogram: expansion of kidney plelois. What is your preliminary diagnosis? +Chronic pyelonephritis -Acute pyelonephritis -Acute glomerulonephritis -Chronic glomerulonephritis -Paranephritis ? A patient K. aged 35 years complains of weakness, dizziness, headache, palpitation, attacks of pain in umbilical, sacral areas, fever up to 38,20C, urine discoloration resembled smoky brown and normal stool. The condition started after bee sting. On clinical examination the jaundice is noted. BP (blood pressure) is 100/60 mm of Hg, spleen is enlarged. RBC ( erythrocytes) are 2,3 * 1012/l, B 72 g/l, K - 0,9, reticulocytes 13,1\%, WBC (leucocytes) 6,2 * 109/l, ESR -25 mm/hour. In urine urobilin reaction is strongly positive, bilirubin pigments are absent. The total bilirubin is 60 mkmol/l, indirect 40 mkmol/l. Serum iron is 20,4 mkmol/l. What clinical diagnosis is the most probable? -Viral hepatitis -Acute liver insufficiency -Iron-deficiency anemia -12- deficiency anemia 34

+Acquired hemolytic anemia ? A 30 years old woman complains of dizziness, weakness, increased nails fragility, alopecia and pica. On clinical examination body t is 36,7, the skin is pale, RR (respiration rate) is 20 per min; Ps 98, BP (blood pressure) 100/60 mm of Hg. In CBC erythrocytes are 2,8*1012/l, Hemoglobin (Hb) 85 g/l, KP-0,9, reticulocytes-2\%; WBC (leucocytes) 4,7*109/l; eosinophils-2\%; stub neutrophils-3\%, segmented neutrophils-62\%, lymphacytes-26\%, monocytes-5\%, ESR-20mm/hour. Serum iron is 6,8 mkmol/l. What preparation is necessary to prescribe to a patient? -Erythrocytal mass -Fresh blood +Iron per os -Iron pareuterally -Vitamin B12-intramuscularly ? A patient aged 43 years was admitted to the hospital. She has been suffering from the bronchial asthma for 7 years. The main clinical features are following. The patient`s position is orthopnoic and she speaks only with separate words. The additional muscles take part in the act of respiration. Respiratory rate is 32 per min. Wheezes are heard in all lungs surface. The heart rate 125 per min. Expiratory peak volume after short acting B2 antagonists intake is 55\% from the normal value. PaO2 is 58 mm of Hg, PaCO2 42 mm Hg, SaO2 90\%. Haw should be classified the bronchial asthma observed in this case? +sever -Moderate to severe -mild -Moderate -Life threatening ? A 75-year-old male complains of headache. On one occasion he transiently lost vision in his right eye. He also complains of aching in the shoulders and neck. There are no focal neurologic findings. Carotid pulses are normal without bruits. Laboratory data show a mild anemia. Erythrocyte sedimation rate is 85. Which of the following is the best approach to management? +Begin glucocorticoid therapy and arrange for temporal artery biopsy -Schedule biopsy and begin corticosteroids based on biopsy results and clinical course -Schedule carotid angiography -Follow ESR and consider further studies if it remains elevated -Start aspirin and defer any invasive studies unless further symptoms develop ? A 55-year-old man with psoriasis has been troubled by long-standing destructive arthritis involving the hands, wrists, shoulders, knees, and ankles. Hand films demonstrate pencil-in-cup deformities. He has been treated with naproxen 1 gm BID, sulfasalazine 1 g bid, prednisone 5 mg qd, and methotrexate 17.5 mg once a week without substantive improvement. Which of the following treatments is most likely to provide long-term benefit? -Cyclophosphamide -Addition of folic acid supplementation -Oral cyclosporine +Tumor necrosis factor inhibitor -Higher-dose steroids in the range of 20 mg of prednisone per day ? A 65-year-old male develops the onset of severe knee pain over 24 hours. The knee is red, swollen, and tender. He has a history of diabetes mellitus and cardiomyopathy. An x-ray of the knee shows linear calcification. Definitive diagnosis is best made by which of the following? -Serum uric acid -Serum calcium +Arthrocentesis and identification of positively birefringent rhomboid crystals -Rheumatoid factor 35

? A 19-year-old male complains of low back morning stiffness, pain, and limitation of motion of shoulders. He has eye pain and photophobia. Diastolic murmur is present on physical exam. Diagnosis? -Behets syndrome +Ankylosing spondylitis -Polymyalgia rheumatica -Reiters syndrome -Drug-induced lupus erythematosus ? A 40-year-old woman has had increasing fatigue and shortness of breath for years. She is suspected of having pulmonary hypertension based on a chest x-ray that shows right ventricular hypertrophy. Pulmonary embolus is ruled out by spiral CT scan. A right heart catheterization confirms the diagnosis of primary pulmonary hypertension. Which of the following is the best next step in the management of the patient? +Acute drug testing with short-acting pulmonary vasodilators -High-dose nifedipine -Intravenous prostacyclin -Lung transplantation ? A 60-year-old obese male complains of excessive daytime sleepiness. He has been in good health except for mild hypertension. He drinks alcohol in moderation. The patients wife states that he snores at night and awakens frequently. Examination of the oropharynx is normal. Which of the following studies is most appropriate? -EEG to assess stage sleep patterns -Ventilation pattern to detect apnea +Arterial O2 saturation -Polysomnography to include all of the above ? An obese 50-year-old woman complains of insomnia, daytime sleepiness, and fatigue. She is found to have recurrent episodes of arterial desaturation about 30 events per hourwith evidence of obstructive apnea. Which of the following is the treatment of choice for this patient? +Nasal continuous positive airway pressure -Uvulopalatopharyngoplasty -Weight reduction -Tracheostomy ? A 55-year-old woman with long-standing chronic lung disease and episodes of acute bronchitis complains of increasing sputum production, which is now on a daily basis. Sputum is thick, and daily sputum production has dramatically increased over several months. There are flecks of blood in the sputum. The patient has lost 8 lb. Fever and chills are absent, and sputum cultures have not revealed specific pathogens. Chest x-ray shows increased pulmonary markings and honeycombing in the lower lobes. CT scan shows a signet ring sign with markedly dilated bronchi. Which of the following is the most likely cause of the patients symptoms? -Pulmonary tuberculosis -Exacerbation of chronic lung disease -Carcinoma of the lung -Anerboic lung abscess +Bronchiectasis ? A 20-year-old fireman comes to the emergency room complaining of headache and dizziness after helping to put out a garage fire. He does not complain of shortness of breath, and the arterial blood gas shows a normal partial pressure of oxygen. Which of the following is the best first step in the management of this patient? -Begin oxygen therapy -Obtain chest x-ray 36

+Obtain carboxyhemoglobin level -Obtain CT scan ? A 36-year-old male comes with the sensation of a racing heart. His blood pressure is 110/70, respiratory rate normal, and O2 saturation 98%. His ECG shows a narrow QRS complex tachycardia with rate 180, which you correctly diagnose as paroxysmal atrial tachycardia. Which of the following is the initial therapy of choice in this hemodynamically stable patient? +Adenosine 6 mg rapid IV bolus -Verapamil 2.5 to 5 mg IV over 1 to 2 min -Diltiazem 0.25 mg/kg IV over 2 min -Digoxin 0.5 mg IV slowly -Lidocaine 1.5 mg/kg IV bolus ? A 50-year-old construction worker continues to have an elevated blood pressure of 160/95 even after a third agent is added to his antihypertensive regimen. Physical exam is normal, electrolytes are normal, and the patient is taking no over-the-counter medications. Which of the following is the next helpful step for this patient? +Check pill count -Evaluate for Cushing syndrome -Check chest x-ray for coarctation of the aorta -Obtain a renal angiogram -Obtain an adrenal CT scan ? A 35-year-old male complains of substernal chest pain aggravated by inspiration and relieved by sitting up. He has a history of tuberculosis. Lung fields are clear to auscultation, and heart sounds are somewhat distant. Chest x-ray shows an enlarged cardiac silhouette. Which of the following is the best next step in evaluation? -Right lateral decubitus film -Cardiac catheterization +Echocardiogram -Serial ECGs -Thallium stress test ? A 42-year-old female suspected of having acute pericarditis has suddenly developed jugular venous distention and hypotension. The ECG shows electrical alternans. Which of the following is the most likely additional physical finding? -Basilar rales halfway up both posterior lung fields -S3 gallop -Strong apical beat +Pulsus paradoxus ? A 27-year-old alcoholic presents with the following electrolytes: calcium 6.9 mg/dL, albumin 3.5 g/dL, magnesium 0.7 mg/dL, phosphorus 2.0 mg/dL. Which of the following is the most important cause of the hypocalcemia? -Poor dietary intake -Hypoalbuminemia -Decreased parathyroid hormone release due to hypomagnesemia +Decreased end organ response to parathyroid hormone due to hypomagnesemia ? A 47-year-old woman complains of fatigue, weight loss, and itching after taking a hot shower. Physical exam shows plethoric facies and an enlarged spleen, which descends 6 cm below the left costal margin. Her white cell count is 17,000 with a normal differential, the platelet count is 560,000, and hemoglobin is 18.7. Liver enzymes and electrolytes are normal; the serum uric acid level is mildly elevated. The most likely underlying process is which of the following? 37

-Myelodysplastic syndrome +Myeloproliferative syndrome -Paraneoplastic syndrome -Form of Cushings syndrome ? A 78-year-old man presents with a 2-month history of headaches. He recently noted pain at the angle of his jaws when he chews. Diagnosis? -Microscopic polyangiitis -Classic polyarteritis nodosa -Takayasus arteritis -Multiple cholesterol embolization syndrome +Giant cell arteritis ? A 60-year-old male develops acute shortness of breath, tachypnea, and tachycardia while hospitalized for congestive heart failure. On physical exam there is no jugular venous distention and the lungs are clear to auscultation and percussion. There is a loud P2 sound. Examination of the lower extremities shows no edema or tenderness. Which of the following is the most important diagnostic step? -Pulmonary angiogram +Ventilation-perfusion scan or chest CT -D-dimer assay -Venous ultrasound ? An asymptomatic 30-year-old female postdoc was noted by her gynecologist to have a cardiac murmur. She was referred for an echocardiogram, with results reported to her as showing mitral valve prolapse. The patient desires more information and now comes to you. Which of the following is true about her condition? +Displacement of one or both mitral valve leaflets posteriorly into the left atrium occurs during systole -Migration of the systolic click and systolic murmur toward the first heart sound will occur during squatting -Prophylactic beta blocker therapy is indicated -Significant mitral regurgitation is likely to occur (>50% chance) sometime in her life -Restriction of vigorous exercise is advised to reduce the risk of sudden cardiac death ? A 45-year-old moderately obese white woman presents after four episodes of severe epigastric and right upper quadrant pain, each episode lasting 30 to 60 min and accompanied by nausea and vomiting. Her most recent episode was very severe, with the pain radiating to the inferior angle of the scapula. Diagnosis? -Acute diverticulitis -Acute pancreatitis +Acute cholecystitis -Intestinal obstruction -Irritable bowel syndrome ? A 43-year-old female presents with hypertension, edema, hyperlipidemia, and a deep venous thrombosis in her left leg. Which of the following is not necessary to diagnose the nephrotic syndrome? -Edema +Hypertension -24-h urine albumin 3 g -Hyperlipidemia ? A 78-year-old man complains of increasing fatigue and bone pain, especially around the knees and ankles. He has a long-standing anemia with a hemoglobin of 9 to 10 g/dL and MCV of 102. He had not responded to therapeutic trials of iron and vitamin B12, but had been symptomatically stable until the past month. Examination reveals pallor and spleen tip just palpable at the left costal margin. CBC reveals hemoglobin of 8.2 g/dL, but for the first time his 38

platelet count is low (15,000); the white blood cell count is 14,000. What is the likely cause of his worsening anemia? -Folic acid deficiency -Viral infection -Myelofibrosis -Tuberculosis -Acute myeloid leukemia ? A 68-year-old woman who has had a previous open cholecystectomy presents with an 8-h history of cramping periumbilical pain. Each episode of pain lasts 3 to 5 min and then abates. Over several hours she develops nausea, vomiting, and abdominal distension. She has been unable to pass stool or flatus for the past 4 hours. Diagnosis? -Acute diverticulitis -Acute pancreatitis -Acute cholecystitis +Intestinal obstruction -Irritable bowel syndrome

39

You might also like